Howdy, Stranger!

It looks like you're new here. If you want to get involved, click one of these buttons!

Question Regarding Weaken and Strengthen Questions

GUDETAMAGUDETAMA Alum Member
edited October 2018 in Logical Reasoning 114 karma

Hello. Lately I have been having a lot of trouble with grasping what exactly I am supposed to be doing with Weaken and Strengthen Questions. I understand that the basic concept is to find an answer choice that will either weaken or strengthen the existing support structure provided in the Stimulus or more it less or more relevant but despite this I find myself having trouble with finding the correct answer.

Because we have to treat what is said in the answer choices as true what does that mean for the assumptions that you can draw from the answer choices? Are we even supposed to be making any assumptions from the answer choices when doing this and if so do they also hold true? Or is the truth of these assumptions from the answer choice dependent on something else like the information and context provided by the Stimulus? Should we even be making any assumptions at all when it comes to the AC?! Also do we treat all the information in the Stim as true as well?

To make my question a bit more clear I will be using Question 15 from "Weaken Questions Problem Set 5" as an example.

Admin note: Please review the forum rules.

4. Do not post LSAT questions, any copyrighted content, or links to content that infringe on copyright.

PT25.S2.Q15: https://7sage.com/lsat_explanations/lsat-25-section-2-question-15/

Which of the following, if true, most seriously undermines the reasoning above?

Answer Choices (With my Reasoning Below)

A.) Admin note: Deleted
X = I picked this answer choice (but it turned out to be incorrect) because I thought that the argument above assumed that it was only temperature that is capable of causing rhododendrons (R) and crocus (C) to blossom. If it was to some other factor which this AC seems to hint at then wouldn’t it weaken the authors argument that because of the observed incident it can be concluded that it would indicate something about the temperature.

B) Admin note: Deleted
X = The subject of this AC is what many people think about being outdoors which is irrelevant and thus does nothing to weaken the argument above.

C) Admin note: Deleted
X = I feel that this would strengthen the argument because if it is also favorable then it is more likely for this incidence to be indicative of the accuracy of a thermometer reading.

D) Admin note: Deleted
Correct AC = This is my problem with this AC. It says that R “CAN” grow 12 feet tall. Are we supposed to draw an assumption that the R plant in the Stim is infact 12 feet tall. “Can” seems to indicate only a possibility but it doesn’t guarantee anything so then I thought that because we had to draw a assumption that this wouldn’t be a strong enough AC to weaken. Also the phrase “is likely” doesn’t guarantee us anything either, it just indicates an increased chance but not something I would consider to be 100%. That 1% that the air temperature might not differ could possibly lead this AC to be useless in weakening. So I am confused as to how much we can assume to make an AC fit with either strengthen or weaken.

E) Admin note: Deleted
X = Seems irrelevant to me although it has the possibility to weaken if the assumption that the author was using this specific thermometer + would have to assume that he is not working in modern temperature range + also would have to assume that he is observing this phenomenon where the thermometer would be less accurate. So too many assumptions that we are not guaranteed of.

Comments

  • MissChanandlerMissChanandler Alum Member Sage
    3256 karma

    You don't need a guarantee for a weaken question. You just needs something that undermines the conclusion. The conclusion of this argument is like "hey, if these things are true, then you know how accurate your thermometer is" and the correct answer choice is like "well maybe not because in a lot of situations (when the plants are different heights) that's not actually true." You don't need to completely disprove an argument for a weakening question.

Sign In or Register to comment.